LSAT and Law School Admissions Forum

Get expert LSAT preparation and law school admissions advice from PowerScore Test Preparation.

 Administrator
PowerScore Staff
  • PowerScore Staff
  • Posts: 8917
  • Joined: Feb 02, 2011
|
#63962
Complete Question Explanation

Must Be True. The correct answer choice is (A)

In this stimulus, the author uses somewhat convoluted language to express the following conclusion
in the opening sentence of the paragraph:

According to free market principles, a government on the brink of a financial crisis can limit the
extent to which foreign investments and loans can be withdrawn in order to prevent economic
collapse.

This conclusion is followed by a premise (introduced with the premise indicator “after all...”): Even
the right to free speech is limited in some ways. The analogy is as follows: We are guaranteed the
right to free speech, but that does not include the guarantee of the right to do potential damage, such
as the common example of shouting “Fire!” in a crowded theatre.

In the same way that the right to yell “Fire” in a crowded theatre is not protected by the right to free
speech, an unregulated ability for investors and lenders to withdraw funds, which might also cause
serious damage, is not guaranteed by the right to a free market.

The stimulus is followed by a Must Be True question stem, so the correct answer choice will reflect
something that the “argument does.”

Answer choice (A): This is the correct answer choice. The argument in the stimulus shows the
limitations on the principles of the free market through comparison to the limitations on the right to
free speech. Neither, the author argues, dictates rights across the board; both are limited in extreme
situations. The fire example could cause a literal rush that would be unsafe; allowing unfettered
withdrawal by investors and lenders could cause a figurative rush that could lead to disaster of a
different sort.

Answer choice (B): This rather awkwardly worded answer choice claims that the argument presumes
the best explanation of observed facts to be the correct explanation. The author of the stimulus does
not make this presumption, but rather draws the analogy to yelling “Fire” in a crowded theatre, and
the limitations on the right to free speech.

Answer choice (C): Again, each incorrect answer to this Must Be True question stem will fail the
Fact Test, and this incorrect choice is no exception. No experimental results are presented, so this
answer cannot be correct.

Answer choice (D): No flawed explanation is discussed in the stimulus. The author simply points out
a limitation on the rights implicit in a free market, through comparison to the limitation on the rights
guaranteed by the right to free speech.

Answer choice (E): The author does not base the conclusion on an empirical generalization, but on a
comparison to the similar case of the limited right to free speech.
 htngo12
  • Posts: 40
  • Joined: May 19, 2016
|
#27247
HI!

For this Method of Reasoning Question, the argument presents that to prevent economic collapse, it limits the extent the $ from investors and lenders that can be withdrawn, that the government does not violate free-market principles when the nation is on the brink of financial crisis.

It the gives an example that involves free speech that does not include the right to shout "Fire!" in a crowd, analogous to the harm done by the investors and lenders rushing to get their $ money out before everyone else is just as real as the harm from a stampede in a theatre.

Even though I picked (A) as the correct answer because essentially compared the two instances resulting the same type of harm.

I was also tied between answer (E) applies an empirical generalization to reach a conclusion about a particular case, in the testing process.

Could you give me an example of how (E) would be applied to the exam?
 Clay Cooper
PowerScore Staff
  • PowerScore Staff
  • Posts: 241
  • Joined: Jul 03, 2015
|
#27260
Hi htngo,

Thanks for your question.

Sure! And not only will I give you a specific example of what an argument like the one E describes would look like, I will also applaud you for thinking about this method of reasoning question that way; for me, the best way to sort through answer choices on this question type is to ask myself: "What would an argument like the one described in answer choice X look like? Can I come up with an example?" and, then, "Does the example match the stimulus argument?"

Anyway, here is your example:

Bobby's story about being abducted by aliens must be a lie, because every other story of alien abduction that has ever been investigated has turned out to be false.

(conclusion about this particular case, supported by a generalization about similar cases which is derived from empirical evidence)

I hope that helps!
 htngo12
  • Posts: 40
  • Joined: May 19, 2016
|
#27337
Oh I definitely see it now.

I've encountered a particular question some exams ago. Given a particular example and then generalizing based on that example to draw their conclusion.

Thanks!

Get the most out of your LSAT Prep Plus subscription.

Analyze and track your performance with our Testing and Analytics Package.